Đến nội dung

Hình ảnh

Topic về Bất đẳng thức trong Tích phân

* * * * - 6 Bình chọn

  • Please log in to reply
Chủ đề này có 125 trả lời

#101
ablrise

ablrise

    Binh nhì

  • Thành viên
  • 17 Bài viết
Bài 51:Cho hàm liên tục $f:[0,+\infty)\to [0,1]$ thoả mãn $f(x+y)\leq f(x)f(y),\forall x,y\geq 0$.Chứng minh:$$\int_0^x f(t)dt \geq x\sqrt{f(2x)}.\forall x\geq 0$$

Bài viết đã được chỉnh sửa nội dung bởi ablrise: 18-02-2013 - 20:26


#102
phudinhgioihan

phudinhgioihan

    PĐGH$\Leftrightarrow$TDST

  • Biên tập viên
  • 348 Bài viết

Bài 45:
b) Hàm $f(x)$ khả tích trên toạn $[0;1]$ và $\int_0^1 f(x)dx >0$. Chứng minh tồn tại đoạn $[a;b]\subset [0;1]$ mà trên đó $f(x)>0$


Giả sử $\int_0^1 f(x)dx=I>0$

Với $n \in \mathbb{N}^*$, xét phân hoạch $P$ chia $[0;1]$ bởi các điểm $\frac{i}{n}\;, i =\overline{0,n}$, ta có

$$I=\lim_{n \to +\infty} \frac{1}{n}\sum_{i=0}^n f(\frac{i}{n})$$

$$\Leftrightarrow \forall \epsilon>0, \exists n_0>0|\; n>n_0 \Rightarrow \left| \frac{1}{n}\sum_{i=0}^n f(\frac{i}{n}) -I \right| <\epsilon$$

Chọn một $0<\epsilon_0<l$ để cố định, khi đó

$$\exists n_0>0|\; n>n_0 ,\; \left| \frac{1}{n}\sum_{i=0}^n f(\frac{i}{n}) -I \right| <\epsilon_0$$

$$\Rightarrow \exists n_0>0|\; n>n_0 , \frac{1}{n}\sum_{i=0}^n f(\frac{i}{n})>I-\epsilon_0>0$$

Do đó phải tồn tại $i_0 \in \mathbb{N}$ sao cho $f(\frac{i_0}{n})>0$. Do $f$ liên tục trên $[0;1]$ nên tồn tại một $\lambda-$lân cận của $\frac{i_0}{n}$ sao cho $f(x)>0 \;, \forall x \in (\frac{i_0}{n}-\lambda;\frac{i_0}{n}+\lambda)$

Do $\dfrac{i_0}{n} \in [0;1]$ nên $(\frac{i_0}{n}-\lambda;\frac{i_0}{n}+\lambda) \cap [0;1]$ là một đoạn, suy ra đpcm.

Bài viết đã được chỉnh sửa nội dung bởi phudinhgioihan: 18-02-2013 - 20:13

Phủ định của giới hạn Hình đã gửi

Đó duy sáng tạo ! Hình đã gửi


https://phudinhgioihan.wordpress.com/

#103
phudinhgioihan

phudinhgioihan

    PĐGH$\Leftrightarrow$TDST

  • Biên tập viên
  • 348 Bài viết

Thêm một bài nữa nào

Bài 47. [Đặng Thành Nam] Cho $f:\left[ {0,1} \right] \to R $là hàm khả vi liên tục. Đặt $M = \mathop {{\rm{max}}}\limits_{x \in \left[ {0,1} \right]} \left| {f'(x)} \right|$. Chứng minh rằng

$$0 \le \int\limits_0^1 {{f^2}(x)dx} - {\left( {\int\limits_0^1 {f(x)dx} } \right)^2} \le M\left( {\mathop {{\rm{max}}}\limits_{x \in \left[ {0,1} \right]} f(x) - \int\limits_0^1 {f(x)dx} } \right)$$

Bài trên có bề ngoài ''gần giống'' với một bài đã post của bạn phudinhgioihan khi đặc biệt hoá hai tham số $a,b$ :
Bài 48:Cho hàm số $f:[0,1]\to \mathbb{R}$ khả vi liên tục trên miền xác định.
Đặt $M=\displaystyle\max_{x\in [ 0;1]} |f'(x)|,m=\displaystyle\min_{x\in [0;1]} |f'(x)|$.Chứng minh rằng :$$\dfrac{m^2}{12}\leq\displaystyle\int_0^1 f^2(x)dx-\left(\displaystyle\int_0^1 f(x)dx\right)^2 \leq \dfrac{M^2}{12}$$



Chứng minh tương tự thật :D.

Xuất phát từ cách chứng minh BDT C-S tích phân...

Với $x,y \in [0;1]$

$$\int_0^1\int_0^1 (f(x)-f(y))^2dxdy=\int_0^1\int_0^1 \left( f^2(x)+f^2(y)-2f(x)f(y) \right) dxdy$$

$$=\int_0^1\int_0^1 f^2(x)dxdy+\int_0^1\int_0^1 f^2(y)dxdy-2\int_0^1\int_0^1f(x)f(y)dxdy$$

$$=2 \left( \int_0^1 f^2(x)dx -\left( \int_0^1 f(x)dx \right)^2 \right) $$

Vậy $$\int_0^1 f^2(x)dx -\left( \int_0^1 f(x)dx \right)^2=\frac{1}{2} \int_0^1\int_0^1 (f(x)-f(y))^2dxdy $$

Từ đl Lagangre suy ra $ m^2 (x-y)^2 \le (f(x)-f(y)^2 \le M^2 (x-y)^2$

Suy ra $$\frac{m}{2}\int_0^1\int_0^1 (x-y)^2dxdy \le \frac{1}{2} \int_0^1\int_0^1 (f(x)-f(y))^2dxdy \le \frac{M^2}{2} \int_0^1\int_0^1 (x-y)^2dxdy$$

$$\Leftrightarrow \dfrac{m^2}{12} \le \frac{1}{2} \int_0^1\int_0^1 (f(x)-f(y))^2dxdy \le \dfrac{M^2}{12}$$

Xong bài 48, đến bài 47, khó hơn tí...

Ta có $$\int_0^1\int_0^1 (f(x)-f(y))^2dxdy =\int_0^1\int_0^1 | f(x)-f(y)| |f(x)-f(y)| dxdy$$

$$\le M \int_0^1\int_0^1 |x-y| |f(x)-f(y)|dxdy \le M \int_0^1\int_0^1 |f(x)-f(y)|dxdy$$

Từ xa xưa ta có $\max\{f(x);f(y)\} =\dfrac{f(x)+f(y)}{2}+\dfrac{|f(x)-f(y)|}{2}$

Suy ra $|f(x)-f(y)|=2\max\{f(x);f(y)\}-(f(x)+f(y)) \le 2 \max_{[0;1]} f(x)-(f(x)+f(y))$

$$\Rightarrow \int_0^1\int_0^1 (f(x)-f(y))^2dxdy \le M \int_0^1\int_0^1 \left( 2\max_{[0;1]}f(x)-f(x)-f(y) \right)dxdy$$

$$\le 2M\left( \max_{[0;1]}f(x)-\int_0^1f(x)dx \right)$$

Vậy $$\int_0^1 f^2(x)dx-\left(\displaystyle\int_0^1 f(x)dx\right)^2 \le M\left( \max_{[0;1]}f(x)-\int_0^1f(x)dx \right) $$

Bài viết đã được chỉnh sửa nội dung bởi phudinhgioihan: 18-02-2013 - 20:35

Phủ định của giới hạn Hình đã gửi

Đó duy sáng tạo ! Hình đã gửi


https://phudinhgioihan.wordpress.com/

#104
phudinhgioihan

phudinhgioihan

    PĐGH$\Leftrightarrow$TDST

  • Biên tập viên
  • 348 Bài viết

Mình nghĩ để ai đó đưa ra lời giải cho hai bài toán trên, vì bài mình đưa ra có dạng tương tự :lol:

Bài 49. [Đặng Thành Nam] Cho $f(x) \ge 0$ là hàm giảm và $f(x) + xf'(x) \ge 0$ với mọi $x \in \left[ {a,b} \right]$. Chứng minh rằng

$$\int\limits_a^b {x{f^2}(x)dx} \le \frac{{b + a}}{{2\left( {b - a} \right)}}{\left( {\int\limits_a^b {f(x)dx} } \right)^2}$$


Với hai hàm thực $f;g$ cùng liên tục trên $[a;b]$, nếu thêm giả thiết một hàm tăng và hàm còn lại giảm thế thì $\forall x;y \in [a;b]$, $(f(x)-f(y))(g(x)-g(y)) \le 0$

$$\Rightarrow \int_0^1\int_0^1 (f(x)-f(y))(g(x)-g(y))dxdy \le 0$$

$$\Leftrightarrow \int_0^1\int_0^1 \left( f(x)g(x)+f(y)g(y)-f(x)g(y)-f(y)g(x) \right) dxdy \le 0$$
$$\Leftrightarrow 2 \left((b-a) \int_0^1 f(x)g(x)dx-\int_a^b f(x)dx\int_a^b g(x)dx \right) \le 0$$

$$\Leftrightarrow \int_0^1 f(x)g(x)dx \le \dfrac{1}{b-a} \int_a^b f(x)dx\int_a^b g(x)dx$$

Quen nhỉ :D

Trở lại vấn đề, ta thấy $f(x) + xf'(x)=(xf(x))' \ge 0$ nên $xf(x)$ là hàm tăng, suy ra

$$\int_a^b xf^2(x)dx \le \dfrac{1}{b-a} \int_a^bxf(x)dx \int_a^b f(x)dx $$

$$\le \dfrac{1}{(b-a)^2} \int_a^bxdx \left(\int_a^b f(x)dx\right)^2 $$

$$\le \dfrac{b+a}{2(b-a)} \left(\int_a^b f(x)dx\right)^2$$

Phủ định của giới hạn Hình đã gửi

Đó duy sáng tạo ! Hình đã gửi


https://phudinhgioihan.wordpress.com/

#105
phudinhgioihan

phudinhgioihan

    PĐGH$\Leftrightarrow$TDST

  • Biên tập viên
  • 348 Bài viết

Bài 51:Cho hàm liên tục $f:[0,+\infty)\to [0,1]$ thoả mãn $f(x+y)\leq f(x)f(y),\forall x,y\geq 0$.Chứng minh:$$\int_0^x f(t)dt \geq x\sqrt{f(2x)}.\forall x\geq 0$$


Bài này lừa tình khiếp @@

Do $f(x) \in [0;1] \;, \forall x \ge 0$ nên $f(x+y) \le f(x)f(y) \le f(x) \;\;, \forall x,y \ge 0$

Suy ra $f$ nghịch biến trên $[0;+\infty)$ , suy ra $\int_0^x f(t)dt \ge \int_0^x f(x)dt=xf(x)=x \sqrt{f(x)f(x)} \ge x\sqrt{f(2x)} $

Phủ định của giới hạn Hình đã gửi

Đó duy sáng tạo ! Hình đã gửi


https://phudinhgioihan.wordpress.com/

#106
Ispectorgadget

Ispectorgadget

    Nothing

  • Quản lý Toán Phổ thông
  • 2946 Bài viết

Bài 52: Cho $f(x),g(x)$ là các hàm liên tục trên $[a;b]$ và $0<a\le f(x)\le A; 0<b\le g(x)\le B \forall x\in [a;b]$. Chứng minh BĐT G.Polya

$$\frac{(ab+AB)^2}{4abAB}\ge \frac{\int_a^bg^2(x)dx\int_a^bf^2(x)dx}{\left(\int_a^bf(x)g(x) \right )^2}\ge \frac{4abAB}{(ab+AB)^2}$$

Spoiler


Bài viết đã được chỉnh sửa nội dung bởi phudinhgioihan: 15-09-2013 - 18:39

►|| The aim of life is self-development. To realize one's nature perfectly - that is what each of us is here for. ™ ♫


#107
phudinhgioihan

phudinhgioihan

    PĐGH$\Leftrightarrow$TDST

  • Biên tập viên
  • 348 Bài viết

Bài 52: Cho $f(x),g(x)$ là các hàm liên tục trên $[a;b]$ và $0<a\le f(x)\le A; 0<b\le g(x)\le B \forall x\in [a;b]$. Chứng minh BĐT G.Polya

$$\frac{(ab+AB)^2}{4abAB}\ge \frac{\int_a^bg^2(x)dx\int_a^bf^2(x)dx}{\left(\int_a^bf(x)g(x) \right )^2}\ge \frac{4abAB}{(ab+AB)^2}$$

Spoiler

 

 

 

Làm nốt để bắt đầu hâm nóng topic :D

 

Từ giả thiết, dễ có $\dfrac{a}{B}\le \dfrac{f(x)}{g(x)} \le \dfrac{A}{b} $

 

$$\Rightarrow \left( f(x)-\dfrac{a}{B} g(x)\right) \left( f(x)-\dfrac{A}{b} g(x)\right)  \le 0$$

 

$$\Leftrightarrow f^2(x)+\dfrac{aA}{bB}g^2(x)-(\dfrac{a}{B}+\dfrac{A}{b})f(x)g(x) \le 0$$

 

$$\Rightarrow \int_a^b f^2(x)dx+\dfrac{aA}{bB}\int_a^b g^2(x)dx \le (\dfrac{a}{B}+\dfrac{A}{b})\int_a^b f(x)g(x)dx $$

 

Lại có:

 

$$\int_a^b f^2(x)dx+\dfrac{aA}{bB}\int_a^b g^2(x)dx \ge 2\sqrt{\dfrac{aA}{bB}\int_a^b f^2(x)dx \int_a^b g^2(x)dx }$$

 

nên suy ra

 

$$2\sqrt{\dfrac{aA}{bB}\int_a^b f^2(x)dx \int_a^b g^2(x)dx } \le (\dfrac{a}{B}+\dfrac{A}{b})\int_a^b f(x)g(x)dx$$

 

$$\Leftrightarrow 4\dfrac{aA}{bB}\int_a^b f^2(x)dx \int_a^b g^2(x)dx \le \dfrac{(ab+AB)^2}{b^2B^2} \left( \int_a^b f(x)g(x)dx \right)^2$$

 

$$\Leftrightarrow \dfrac{(ab+AB)^2}{4abAB} \ge \dfrac{\int_a^bg^2(x)dx\int_a^bf^2(x)dx}{\left(\int_a^bf(x)g(x) \right )^2}$$

 

Bất đẳng thức

 

 

$$\dfrac{\int_a^bg^2(x)dx\int_a^bf^2(x)dx}{\left(\int_a^bf(x)g(x) \right )^2}\ge \frac{4abAB}{(ab+AB)^2}$$

 

là dễ dàng vì $$\dfrac{\int_a^bg^2(x)dx\int_a^bf^2(x)dx}{\left(\int_a^bf(x)g(x) \right )^2} \ge 1 \ge \dfrac{4abAB}{(ab+AB)^2}$$


Phủ định của giới hạn Hình đã gửi

Đó duy sáng tạo ! Hình đã gửi


https://phudinhgioihan.wordpress.com/

#108
phudinhgioihan

phudinhgioihan

    PĐGH$\Leftrightarrow$TDST

  • Biên tập viên
  • 348 Bài viết

Bài 53: (mới chế :D)

 

a) Cho hai số thực $a<b$ và hàm số $f,g:[a;b]  \to \mathbb{R}$ khả tích trên $[a;b]$ sao cho $\int_a^b f(x)dx \int_a^b g(x)dx=b-a$

 

Chứng minh: $\int_a^b \left(f^2(x)+g^2(x)\right)dx \ge \int_a^b \left(f(x)+g(x) \right)dx $

 

b)  Cho hàm số $f:[0;1] \to \mathbb{R}$ khả tích trên $[0;1]$, chứng minh

 

$$\int_0^1 f^2(x)dx-\int_0^1f(x)dx \ge \dfrac{1}{2} \left[ \left( \int_0^1 f(x)dx \right)^2-1 \right]$$


Phủ định của giới hạn Hình đã gửi

Đó duy sáng tạo ! Hình đã gửi


https://phudinhgioihan.wordpress.com/

#109
dangnamneu

dangnamneu

    Hạ sĩ

  • Thành viên
  • 68 Bài viết


Bài 53: (mới chế :D)

 

b)  Cho hàm số $f:[0;1] \to \mathbb{R}$ khả tích trên $[0;1]$, chứng minh

 

$$\int_0^1 f^2(x)dx-\int_0^1f(x)dx \ge \dfrac{1}{2} \left[ \left( \int_0^1 f(x)dx \right)^2-1 \right]$$


 

 

 

Mình làm câu b) Câu a thử tương tự xem sao hehe

Viết lại bất đẳng thức dưới dạng

${\left( {\int\limits_0^1 {f(x)dx}  + 1} \right)^2} \le 2\left( {\int\limits_0^1 {{f^2}(x)dx}  + 1} \right)$.

Bất đẳng thức trên hiển nhiên đúng do

${\left( {\int\limits_0^1 {f(x)dx}  + 1} \right)^2} \le \left( {{1^2} + {1^2}} \right)\left[ {{{\left( {\int\limits_0^1 {f(x)dx} } \right)}^2} + {1^2}} \right] \le 2\left( {\int\limits_0^1 {{f^2}(x)dx}  + 1} \right)$


Bài viết đã được chỉnh sửa nội dung bởi Ispectorgadget: 16-09-2013 - 18:14

Giáo viên môn Toán tại website : http://vted.vn


#110
phudinhgioihan

phudinhgioihan

    PĐGH$\Leftrightarrow$TDST

  • Biên tập viên
  • 348 Bài viết

Bài 54:

 

Cho $f:[0;1] \to \mathbb{R} $ đồng biến, khả tích trên $[0;1]$ và $f(0) \ge 0$. Chứng minh

 

$$\left(\int_0^1 x^4f(x)dx \right)^2 \le \int_0^1 x^3f(x) dx \int_0^1 x^5f(x)dx \le \dfrac{25}{24} \left( \int_0^1 x^4f(x)dx \right)^2 $$


Phủ định của giới hạn Hình đã gửi

Đó duy sáng tạo ! Hình đã gửi


https://phudinhgioihan.wordpress.com/

#111
phudinhgioihan

phudinhgioihan

    PĐGH$\Leftrightarrow$TDST

  • Biên tập viên
  • 348 Bài viết

Bài 55: Cho $D$ là tập hợp các hàm $f$ biến thực khả vi liên tục trên $[0;1]$ và $f(0)=0\;, f(1)=1 $

 

Đặt $I(f)=\int_0^1 (1+x^2)(f'(x))^2dx $

 

Tính  $\min_{f \in D} I(f) $

 

 


Phủ định của giới hạn Hình đã gửi

Đó duy sáng tạo ! Hình đã gửi


https://phudinhgioihan.wordpress.com/

#112
babymath

babymath

    Binh nhất

  • Thành viên
  • 33 Bài viết
Mới tìm được một bài trên mạng
Bài 56 Giả sử $f(x)$ là hàm liên tục trên $[-1,\ 1]$ thoả mãn các điều kiện:
 
1) $f(-1)\geq f(1).$
 
2) $x+f(x)$ là hàm không giảm.
 
3) $\int_{-1}^ 1 f(x)\ dx=0.$
 
Chứng minh $\int_{-1}^1 f(x)^2dx\leq \frac 23.$


#113
1414141

1414141

    Trung sĩ

  • Thành viên
  • 132 Bài viết

Bài 57: Cho $f(x)$ xác định và liên tục trên [0,1] thỏa mãn $0 \le f(x) \le 1$. Chứng minh rằng

$$ \int_{0}^1 f(x) dx \ge \bigg(\int_0^1f(x^{2013})dx\bigg)^{2013}$$


Tôi đang thay đổi !

#114
happyfree

happyfree

    Trung sĩ

  • Thành viên
  • 123 Bài viết

Bài 58 :cho $f$ liên tục;có đạo hàm $f'$ liên tục trên đoạn [0;1] và $f(0)=0$.

cmr: $\int_{0}^{1}|f(x)f'(x)|dx\leq\frac{1}{2}\int_{0}^{1}|f'(x)|^{2}dx$


Bài viết đã được chỉnh sửa nội dung bởi happyfree: 09-08-2015 - 22:34


#115
mainayemsethanhanhChau

mainayemsethanhanhChau

    Lính mới

  • Thành viên
  • 4 Bài viết

Bài 59: $f : [0,1] \to \mathbb{R}$ là một hàm khả vi với $f(1) = 0$ và đạo hàm của chúng liên tục. Chứng minh rằng $\int^1_0 (f'(x))^2\,dx \ge 3(\int^1_0 f(x)\,dx)^2$ 


Bài viết đã được chỉnh sửa nội dung bởi mainayemsethanhanhChau: 23-09-2015 - 12:47


#116
leehaan

leehaan

    Lính mới

  • Thành viên mới
  • 3 Bài viết

các bác hộ e với :) 

Hình gửi kèm

  • Capture.PNG


#117
WhjteShadow

WhjteShadow

    Thượng úy

  • Phó Quản lý Toán Ứng dụ
  • 1323 Bài viết

các bác hộ e với :)

Vế phải trong bất đẳng thức của bạn không biết có nhầm lẫn gì không ?


“There is no way home, home is the way.” - Thich Nhat Hanh

#118
Mrnhan

Mrnhan

    $\text{Uchiha Itachi}$

  • Thành viên
  • 1100 Bài viết

Vế phải trong bất đẳng thức của bạn không biết có nhầm lẫn gì không ?

 

Hình như bạn ấy sai điều kiện thôi, điều kiện phải là $f(0)=0$

 

Nếu thế thì lời giả sẽ là:

 

$$f(x)=\int_{0}^{x}f'(t)dt\leq \left ( \int_{0}^{x}1^2 \right )^{1/2} \left( \int_{0}^{x}\left ( f'(t) \right )^2 \right )^{1/2}$$

 

$$\Rightarrow f^2(x) \leq x \int_{0}^{x}\left ( f'(t) \right )^2dt\leq x \int_{0}^{1}\left ( f'(t) \right )^2dt$$

 

$$\Rightarrow \int_{0}^{1} f^2(x)dx\leq \int_{0}^{1}xdx\int_{0}^{1}\left ( f'(t) \right )^2dt=\frac{1}{2}\int_{0}^{1}\left ( f'(x) \right )^2dx$$


$\text{Cứ làm việc chăm chỉ trong im lặng}$

Hình đã gửi$\text{Hãy để thành công trở thành tiếng nói của bạn}$Hình đã gửi


#119
leehaan

leehaan

    Lính mới

  • Thành viên mới
  • 3 Bài viết

Hình như bạn ấy sai điều kiện thôi, điều kiện phải là $f(0)=0$

 

Nếu thế thì lời giả sẽ là:

 

$$f(x)=\int_{0}^{x}f'(t)dt\leq \left ( \int_{0}^{x}1^2 \right )^{1/2} \left( \int_{0}^{x}\left ( f'(t) \right )^2 \right )^{1/2}$$

 

$$\Rightarrow f^2(x) \leq x \int_{0}^{x}\left ( f'(t) \right )^2dt\leq x \int_{0}^{1}\left ( f'(t) \right )^2dt$$

 

$$\Rightarrow \int_{0}^{1} f^2(x)dx\leq \int_{0}^{1}xdx\int_{0}^{1}\left ( f'(t) \right )^2dt=\frac{1}{2}\int_{0}^{1}\left ( f'(x) \right )^2dx$$

đề không sai đâu bạn.
mình cũng làm ntn mà không ra.



#120
leehaan

leehaan

    Lính mới

  • Thành viên mới
  • 3 Bài viết

đề không sai đâu bạn mình cũng thử làm như bạn rồi mà không ra 






0 người đang xem chủ đề

0 thành viên, 0 khách, 0 thành viên ẩn danh